Скобки Пуассона и магнитное поле [закрыто]

Я учусь на математике, пытаюсь выучить физику, поэтому извините, если я упустил что-то простое. Я думаю, что основная проблема заключается в отсутствии опыта работы с символом Леви-Чевита.

У нас есть частица в магнитном поле Б "=" × А , описываемый гамильтонианом (взяв с "=" 1 ):

ЧАС "=" 1 2 м ( п е А ( р ) ) 2 "=" м 2 р ˙ 2

Мне сказали, что структура скобки Пуассона читается { м р ˙ а , м р ˙ б } "=" е ϵ а б с Б с , и попытались доказать это следующим образом:

{ м р ˙ а , м р ˙ б } "=" { п а е А а ( р ) , п б е А б ( р ) }                                                                                                       "=" { п а , п б } е { п а , А б ( р ) } + е { п б , А а ( р ) } + е 2 { А а ( р ) , А б ( р ) }

Теперь первый и последний члены здесь равны нулю, поэтому это упрощается до:

{ м р ˙ а , м р ˙ б } "=" е А б ( р ) р а е А а ( р ) р б

используя тот факт, что { п а , ф ( р ) } "=" ф ( р ) р а . Я не слишком уверен, куда идти дальше, я предполагаю, что нужна какая-то манипуляция с символом Леви-Чивиты? Заранее спасибо.

Кто-нибудь может пояснить, почему это не по теме? @jl2 «спросите [ред] о конкретной концепции физики» и «покажите [ред] некоторые усилия для решения проблемы», нет? Я думаю, что он прошел 90% пути, и ему просто нужно было несколько указателей на символ Леви-Чевита, как он сказал. Думаю, я мог бы остановиться на несколько шагов в своем ответе, позволив ему закончить? Тем не менее, это касается моего ответа, а не его вопроса.
Извините, действительно смущен этим. Я не понимаю, почему вопрос неконкретен или как я не приложил никаких усилий для достижения решения? Можно ли как-то вернуть ответ? Ничто в Справочном центре, похоже, не указывает на то, что с этим что-то не так.

Ответы (1)

Я собираюсь везде использовать обозначение суммирования Эйнштейна .

Ты почти там. Вам просто нужно использовать

Б "=" × А
или, что то же самое,
Б к "=" р я А Дж ϵ я Дж к "=" 1 2 ( р я А Дж ϵ я Дж к + р Дж А я ϵ Дж я к ) "=" 1 2 ( р я А Дж р Дж А я ) ϵ я Дж к
Там я использовал антисимметрию ϵ я Дж к при обмене любых двух индексов.

Теперь, используя первое уравнение здесь ,

Б к ϵ к р с "=" 1 2 ( р я А Дж р Дж А я ) ϵ к я Дж ϵ к р с "=" 1 2 ( р я А Дж р Дж А я ) ( дельта я р дельта Дж с дельта я с дельта Дж р ) "=" 1 2 ( р р А с р с А р ) 1 2 ( р с А р р р А с ) "=" р р А с р с А р